Line 53: Line 53:
 
<math>\color{blue}\text{Solution 2:}</math>  
 
<math>\color{blue}\text{Solution 2:}</math>  
  
One of the basic feasible solution is an optimal solution.<br><font face="serif"></font><span class="texhtml">The equality constraints can be represented in the form ''A''''x'' = ''b'',</span>  
+
One of the basic feasible solution is an optimal solution.<br><font face="serif"></font><span class="texhtml">The equality constraints can be represented in the form ''A'''x'''''<b> = ''b'',</b></span>  
  
 
<math>A=\begin{bmatrix}
 
<math>A=\begin{bmatrix}
Line 67: Line 67:
 
\end{bmatrix}</math>  
 
\end{bmatrix}</math>  
  
<math>\text{The fist basis candidate is } \begin{pmatrix}
+
<math>\text{The first basis candidate is } \begin{pmatrix}
 
a_{1} &  
 
a_{1} &  
 
a_{2}  
 
a_{2}  
Line 82: Line 82:
 
<math>x^{\left( 1 \right)}= \begin{bmatrix}
 
<math>x^{\left( 1 \right)}= \begin{bmatrix}
 
-3 & 4 & 0
 
-3 & 4 & 0
\end{bmatrix} \text{ is BFS. } f_{1}=-9</math>  
+
\end{bmatrix}^{T} \text{ is BFS. } f_{1}=-9</math>  
  
 
<math>\text{The second basis candidate is } \begin{pmatrix}
 
<math>\text{The second basis candidate is } \begin{pmatrix}
Line 99: Line 99:
 
<math>x^{\left( 2 \right)}= \begin{bmatrix}
 
<math>x^{\left( 2 \right)}= \begin{bmatrix}
 
0 & 1 & -3
 
0 & 1 & -3
\end{bmatrix} \text{ is BFS. } f_{2}=-15</math>  
+
\end{bmatrix}^{T} \text{ is BFS. } f_{2}=-15</math>  
  
 
<math>\text{The third basis candidate is } \begin{pmatrix}
 
<math>\text{The third basis candidate is } \begin{pmatrix}
Line 116: Line 116:
 
<math>x^{\left( 2 \right)}= \begin{bmatrix}
 
<math>x^{\left( 2 \right)}= \begin{bmatrix}
 
1 & 0 & -5
 
1 & 0 & -5
\end{bmatrix} \text{ is BFS. } f_{3}=-21</math>  
+
\end{bmatrix}^{T} \text{ is BFS. } f_{3}=-21</math>  
  
 
<math>\because f_{1}>f_{2}>f_{3}</math>  
 
<math>\because f_{1}>f_{2}>f_{3}</math>  
Line 138: Line 138:
 
<math>\color{blue}\text{Solution}</math>  
 
<math>\color{blue}\text{Solution}</math>  
  
<math>\text{The equality constraints can be represented in the form } Ax=b,</math><br>  
+
<span class="texhtml">The equality constraints can be represented in the form ''A''''x'' = ''b'',</span><br>  
  
 
<math>A= \begin{bmatrix}
 
<math>A= \begin{bmatrix}
Line 150: Line 150:
 
4\\  
 
4\\  
 
2
 
2
\end{bmatrix}</math>
+
\end{bmatrix}</math>  
  
For basis candidate&nbsp;
+
<math>\text{The fist basis candidate is } \begin{pmatrix}
 +
a_{1} &  
 +
a_{2}
 +
\end{pmatrix}</math><br>
  
<br>  
+
<math>\text{The corresponding basic solution is } x^{\left( 1 \right)}= \begin{bmatrix}
 +
4 & 2 & 0
 +
\end{bmatrix}^{T} \text{ is BFS. The objective function} f_{1}=14</math><br>
 +
 
 +
<math>\text{The fist basis candidate is } \begin{pmatrix}
 +
a_{1} &
 +
a_{3}
 +
\end{pmatrix}</math><br>
 +
 
 +
<math>\text{The fist basis candidate is } \begin{pmatrix}
 +
a_{1} &
 +
a_{3}
 +
\end{pmatrix}</math>
  
 
<br>  
 
<br>  

Revision as of 21:23, 28 June 2012

ECE Ph.D. Qualifying Exam in "Automatic Control" (AC)

Question 3, Part 3, August 2011

Part 1,2,3,4,5

 $ \color{blue}\text{3. } \left( \text{20 pts} \right) \text{ Solve the following linear program, } $

maximize    − x1 − 3x2 + 4x3

subject to    x1 + 2x2x3 = 5

                   2x1 + 3x2x3 = 6

                   $ x_{1} \text{ free, } x_{2}\geq0, x_{3}\leq0. $


The Fundamental Theorem of Linear Programming that one of the basic feasible solutions is an optimal solution. So we generate all possible basic feasible solutions and select from them the optimal one.


$ \color{blue}\text{Solution 1:} $

$ \left.\begin{matrix} x_{1}+2x_{2}-x_{3}=5 \\ 2x_{1}+3x_{2}-x_{3}=6 \end{matrix}\right\}\Rightarrow x_{1}=5-2x_{2}+x_{3}=3-\frac{3}{2}x_{2}+\frac{1}{2}x_{3} $

$ \Rightarrow x_{2}-x_{3}=4 $

$ \text{It is equivalent to min } x_{1}+3x_{2}-4x_{3} =5-2x_{2}+x_{3}+3x_{2}-4x_{3} = x_{2}-3x_{3}+5, x_{2}\geq0, x_{3}\leq0 $

$ x_{2}-3x_{3}+5 = x_{2}-x_{3}-2x_{3}+5=9-2x_{3}\geq9 $   $ \color{green} \text{constrain: } x_{3}\leq0 \Rightarrow x_{3}=0 $

$ \text{Equivalently, } -x_{1}-3x_{2}+4x_{3}\leq-9 $

$ \text{Equality is satisfied when } x_{3}=0, x_{2} =4+0=4, x_{1}=5-2\times4=-3 $

$ \Rightarrow \left\{\begin{matrix} x_{1}=-3\\ x_{2}=4\\ x_{3}=0 \end{matrix}\right. $

$ \color{green} \text{This solution is not using the Fundamental Theorem of Linear Programming} $


$ \color{blue}\text{Solution 2:} $

One of the basic feasible solution is an optimal solution.
The equality constraints can be represented in the form Ax = b,

$ A=\begin{bmatrix} 1 & 2 & -1\\ 2 & 3 & -1 \end{bmatrix}=\begin{bmatrix} a_{1}& a_{2}& a_{3} \end{bmatrix}; b=\begin{bmatrix} 5\\ 6 \end{bmatrix} $

$ \text{The first basis candidate is } \begin{pmatrix} a_{1} & a_{2} \end{pmatrix} $

$ \left [ A|b \right ] =\begin{bmatrix} 1 & 2 & -1 & 5 \\ 2 & 3 & -1 & 6 \end{bmatrix} = \cdots = \begin{bmatrix} 1 & 0 & 1 & -3 \\ 0 & 1 & -1 & 4 \end{bmatrix} $

$ x^{\left( 1 \right)}= \begin{bmatrix} -3 & 4 & 0 \end{bmatrix}^{T} \text{ is BFS. } f_{1}=-9 $

$ \text{The second basis candidate is } \begin{pmatrix} a_{2} & a_{3} \end{pmatrix} $

$ \left [ A|b \right ] =\begin{bmatrix} 1 & 2 & -1 & 5 \\ 2 & 3 & -1 & 6 \end{bmatrix} = \cdots = \begin{bmatrix} 1 & 0 & 1 & -3 \\ 1 & 1 & 0 & 1 \end{bmatrix} $

$ x^{\left( 2 \right)}= \begin{bmatrix} 0 & 1 & -3 \end{bmatrix}^{T} \text{ is BFS. } f_{2}=-15 $

$ \text{The third basis candidate is } \begin{pmatrix} a_{1} & a_{3} \end{pmatrix} $

$ \left [ A|b \right ] =\begin{bmatrix} 1 & 2 & -1 & 5 \\ 2 & 3 & -1 & 6 \end{bmatrix} = \cdots = \begin{bmatrix} 0 & 1 & 1 & 4 \\ 1 & 1 & 0 & 1 \end{bmatrix} $

$ x^{\left( 2 \right)}= \begin{bmatrix} 1 & 0 & -5 \end{bmatrix}^{T} \text{ is BFS. } f_{3}=-21 $

$ \because f_{1}>f_{2}>f_{3} $

$ \therefore \text{The optimal solution is } x^{*}=\begin{bmatrix} -3 & 4 & 0 \end{bmatrix} \text{ with objective value } -9 $


$ \color{blue} \text{Related Problem: Solve following linear programming problem,} $

minimize   3x1 + x2 + x3

subject to  x1 + x3 = 4

                  x2x3 = 2

                  $ x_{1}\geq0,x_{2}\geq0,x_{3}\geq0, $

$ \color{blue}\text{Solution} $

The equality constraints can be represented in the form A'x = b,

$ A= \begin{bmatrix} 1 & 0 & 1\\ 0 & 1 & -1 \end{bmatrix}=\begin{bmatrix} a_{1}& a_{2}& a_{3} \end{bmatrix}; b=\begin{bmatrix} 4\\ 2 \end{bmatrix} $

$ \text{The fist basis candidate is } \begin{pmatrix} a_{1} & a_{2} \end{pmatrix} $

$ \text{The corresponding basic solution is } x^{\left( 1 \right)}= \begin{bmatrix} 4 & 2 & 0 \end{bmatrix}^{T} \text{ is BFS. The objective function} f_{1}=14 $

$ \text{The fist basis candidate is } \begin{pmatrix} a_{1} & a_{3} \end{pmatrix} $

$ \text{The fist basis candidate is } \begin{pmatrix} a_{1} & a_{3} \end{pmatrix} $




Automatic Control (AC)- Question 3, August 2011

Go to



Back to ECE Qualifying Exams (QE) page

Alumni Liaison

Correspondence Chess Grandmaster and Purdue Alumni

Prof. Dan Fleetwood